0 Daumen
245 Aufrufe

Zeigen Sie mittels der Definition des Grenzwertes, dass

\( \lim \limits_{n \rightarrow \infty} \frac{4 n-1}{3 n+2}=\frac{4}{3} \)
gilt, d. h. zeigen Sie: Zu jedem \( \varepsilon>0 \) gibt es ein \( N(\varepsilon) \in \mathbb{N} \), so dass für alle \( n \geq N(\varepsilon) \)

\( \left|\frac{4 n-1}{3 n+2}-\frac{4}{3}\right|<\varepsilon \)

erfüllt ist.

Geben Sie ein derartiges \( N(\varepsilon) \) für \( \varepsilon \in\left\{\frac{1}{30}, \frac{1}{100}, \frac{1}{300}\right\} \) an.

Avatar von

2 Antworten

+1 Daumen
 
Beste Antwort

Aloha :)

Wir vermuten als Grenzwert der Folge \(a_n=\frac{4n-1}{3n+2}\) den Wert \(a=\frac43\).$$\left|a_n-a\right|=\left|\frac{4n-1}{3n+2}-\frac43\right|=\left|\frac{3(4n-1)-4(3n+2)}{3(3n+2)}\right|=\left|\frac{12n-3-12n-8}{9n+6}\right|=\frac{11}{9n+6}$$

Wir wählen nun ein \(\varepsilon>0\) beliebig aber fest und prüfen, ob es für fast alle \(n\), d.h. ab einem bestimmenten \(n_0\), gößer als unser Ergebnis ist.

Wenn wir den Nenner eines postiven Bruchs verkleinern, wird der Bruch größer, daher gilt:$$|a_n-a|<\frac{11}{9n}\stackrel{!}{<}\varepsilon\implies\frac{9n}{11}>\frac1\varepsilon\implies n>\frac{11}{9\varepsilon}\implies n_0=\left\lceil\frac{11}{9\varepsilon}\right\rceil$$

Für alle \(\varepsilon>0\) gibt es also ein \(n_0\), sodass für alle \(n\ge n_0\) gilt: \(|a_n-a|<\varepsilon\).

Damit konvergiert die Folge \((a_n)\to a=\frac43\).

Jetzt sollst du noch die \(n_0(\varepsilon)\) für die 3 angegebenen \(\varepsilon\)-Werte bestimmen, also einfach nur einsetzen.

Avatar von 149 k 🚀
0 Daumen

hallo

da der Ausdruck int Betrag immer negativ ist (warum)kennt du einfach das negative des Betrags nehmen <ε  oder =ε und n ausrechen und das nächst größere ganze n-

Gruß  lul

Avatar von 107 k 🚀

Ein anderes Problem?

Stell deine Frage

Keine ähnlichen Fragen gefunden

Willkommen bei der Mathelounge! Stell deine Frage einfach und kostenlos

x
Made by a lovely community